Minimum and maximum of an equation

  • Thread starter Thread starter Gear300
  • Start date Start date
  • Tags Tags
    Maximum Minimum
Click For Summary
The discussion revolves around confirming the mathematical derivation of the equation A + B - C = D, where B is a function of A and C is constant. The derivative dD/dA is calculated as 1 + dB/dA, leading to the condition for finding min/max points where dD/dA equals zero. The conclusion drawn is that for the minimum/maximum of D, B must equal -A, resulting in Dmin/max being equal to -C. Additionally, it is clarified that if dB/dA is -1, B can be expressed as -A plus a constant, indicating a linear relationship. The mathematics presented is affirmed as correct, with a focus on the implications of the linearity of B.
Gear300
Messages
1,209
Reaction score
9
The problem here is relatively simple...I just need to confirm something:

A + B - C = D, in which B is a function of A and C is constant.
dD/dA = 1 + dB/dA for the derivative
To find a min/max of D for certain angles of A,
dD/dA = 0 = 1 + dB/dA
dB/dA = -1
dB = -dA
B = -A, which would imply that D is a min/max where B = -A and dB/dA = -1
Dmin/max = A - A - C
Dmin/max = -C, and thus D is a min/max where it is equivalent to -C.

Was my mathematics wrong anywhere in there?
 
Physics news on Phys.org


Gear300 said:
The problem here is relatively simple...I just need to confirm something:

A + B - C = D, in which B is a function of A and C is constant.
dD/dA = 1 + dB/dA for the derivative
To find a min/max of D for certain angles of A,
dD/dA = 0 = 1 + dB/dA
dB/dA = -1
dB = -dA
B = -A, which would imply that D is a min/max where B = -A and dB/dA = -1
Dmin/max = A - A - C
Dmin/max = -C, and thus D is a min/max where it is equivalent to -C.

Was my mathematics wrong anywhere in there?

If dB/dA = -1, then B = -A + a constant. IOW, the graph of B is a straight line with slope -1. Also, dD/dA = -A + a different constant.
 


I see...thank you
 
Question: A clock's minute hand has length 4 and its hour hand has length 3. What is the distance between the tips at the moment when it is increasing most rapidly?(Putnam Exam Question) Answer: Making assumption that both the hands moves at constant angular velocities, the answer is ## \sqrt{7} .## But don't you think this assumption is somewhat doubtful and wrong?

Similar threads

Replies
3
Views
2K
  • · Replies 5 ·
Replies
5
Views
3K
  • · Replies 5 ·
Replies
5
Views
2K
  • · Replies 1 ·
Replies
1
Views
3K
  • · Replies 1 ·
Replies
1
Views
4K
  • · Replies 1 ·
Replies
1
Views
1K
  • · Replies 5 ·
Replies
5
Views
2K
  • · Replies 1 ·
Replies
1
Views
1K
  • · Replies 2 ·
Replies
2
Views
3K
  • · Replies 4 ·
Replies
4
Views
3K